Difference between revisions of "Mock AIME 2 2006-2007 Problems/Problem 5"

m
Line 2: Line 2:
  
 
Given that <math>\displaystyle  iz^2=1+\frac 2z + \frac{3}{z^2}+\frac{4}{z ^3}+\frac{5}{z^4}+\cdots</math> and <math>\displaystyle z=n\pm \sqrt{-i},</math> find <math>\displaystyle  \lfloor 100n \rfloor.</math>
 
Given that <math>\displaystyle  iz^2=1+\frac 2z + \frac{3}{z^2}+\frac{4}{z ^3}+\frac{5}{z^4}+\cdots</math> and <math>\displaystyle z=n\pm \sqrt{-i},</math> find <math>\displaystyle  \lfloor 100n \rfloor.</math>
 +
 +
==Solution==
 +
{{solution}}
 +
 +
----
 +
 +
*[[Mock AIME 2 2006-2007/Problem 4 | Previous Problem]]
 +
 +
*[[Mock AIME 2 2006-2007/Problem 6 | Next Problem]]
 +
 +
*[[Mock AIME 2 2006-2007]]

Revision as of 19:47, 22 August 2006

Problem

Given that $\displaystyle  iz^2=1+\frac 2z + \frac{3}{z^2}+\frac{4}{z ^3}+\frac{5}{z^4}+\cdots$ and $\displaystyle z=n\pm \sqrt{-i},$ find $\displaystyle  \lfloor 100n \rfloor.$

Solution

This problem needs a solution. If you have a solution for it, please help us out by adding it.